Answer & Explanation for G.S. Test-8 held on 18th March 2018

1. Consider the following Padma Awardees and their field of work:

Which of the above is/are correctly matched? (a) 1 only (b) 1 and 2 only (c) 2 and 3 only (d) 1, 2 and 3

Answer: (a) Explanation: Padma Shri Awardees Their work/field 1. Nouf Marwaai : 1st certified yoga instructor in Saudi Arabia 2. Murlikant Petkar : ’s 1st Paralympics Gold Medallist at the 1972 Summer Paralympics in Germany. 3. Rajgopalan Vasudevan : Developed and patented an innovative method to reuse plastic waste to construct roads

2. With reference to the river Mahanadi, consider the following statements: (1) The source of Mahanadi is in the hills of central Chhattisgarh, from where it first flows northwards. (2) After joining Seonath river (its longest tributary), Mahanadi turns eastwards. (3) Cuttack lies on the banks of Mahanadi. Which of the statements given above are correct? (a) 1 and 2 only (b) 2 and 3 only (c) 1 and 3 only (d) 1, 2 and 3

Answer: (d) Explanation:  The 858 kilometres long Mahanadi river flows through the states of Chhattisgarh and Odisha. It gets formed by numerous mountain streams and the farthest headstream is located in hills of the Dhamtari district of Chhattisgarh. These hills are extensions of the Eastern Ghats and are a source of many other streams which then go on to join the Mahanadi.  The river first flows to the north and drains Raipur. After Bilaspur, it is joined by its major tributary river Seonath. After that the river flows eastward and is joined by Jonk, Hasdeo rivers and reaches Odisha. Near Sambalpur of Odisha, the largest dam of the world Hirakud dam blocks its water.  After crossing the Satkosia Gorge, it meets the plains of Odisha and reaches Cuttack.

3. Consider the following statements regarding the Buddhist Councils: (1) The first council was conducted under the patronage of King Ajatashatru of Haryanka dynasty in Rajgriha. (2) The third council was conducted under the patronage of Emperor Ashoka of Maurya dynasty in 250 BC. Which of the statements given above is/are correct? (a) 1 only 1

(b) 2 only (c) Both 1 and 2 (d) Neither 1 nor 2

Answer: (c) Explanation: Council Year Ruler (Dynasty) Place 1 483 BC Ajatashatru (Haryanka) Rajgriha 2 383 BC Kalasoka (Sisunaga) Vaishali 3 250 BC Ashoka (Maurya) Pataliputra 4 1 AD Kanishka (Kushan) Kundalvan (Kashmir)

4. Consider the following statements: (1) The Supreme Court, in a recent judgement, has criminalised sex with a minor wife aged between 15 and 18 years. (2) The Protection of Women from Domestic Violence Act, 2005 is civil in nature but criminal in procedure and offence under this Act is cognisable and non-bailable. Which of the statements given above is/are correct? (a) 1 only (b) 2 only (c) Both 1 and 2 (d) Neither 1 nor 2

Answer: (c) Explanation:  The Supreme Court, in a landmark judgement, has criminalised sex with a minor wife aged between 15 and 18 years, saying the exception (2 to Section 375, which defines rape, of the IPC, amended by the Criminal Law (Amendment) Act, 2013, which allowed such a sexual act) in the rape law was arbitrary and violates the Constitution. The age of consent has been made 18 from 15 in these cases.  The Protection of Women from Domestic Violence Act, 2005 provides for the first time in Indian law a definition of "domestic violence", with this definition being broad and including not only physical violence, but also other forms of violence such as emotional/verbal, sexual, and economic abuse. This Act is essentially a civil law, but the legislation has prescribed that courts have to proceed in such cases as per the Criminal Procedure Code (CrPC) for the purpose of effective actions and offence under this Act is cognisable and non-bailable.

5. Consider the following Awards and Awardees:

Awardees Awards 1. Dr. Vinod Paul Ihsan Doğramacı Family Health Foundation Prize by World Health Organisation (WHO). 2. Soumitra Scientific and Engineering Academy Award at Oscars Scientific and Chatterjee Technical Awards 2018 3. Vikas Sathaye France’s highest civilian award, Legion d’ Honneur.

Which of the above pairs is/are correctly matched? (a) 1 only (b) 1 and 2 only (c) 2 and 3 only (d) 1, 2 and 3

Answer: (a) Explanation:

Awardees Awards

2

Dr. Vinod Paul Ihsan Doğramacı Family Health Foundation Prize by World Health Organisation (WHO). Soumitra Scientific and Engineering Academy Award at Oscars Scientific and Chatterjee Technical Awards 2018 Vikas Sathaye France’s highest civilian award, Legion d’ Honneur.

6. Consider the following islands: (1) Barren Island (2) Narcondam Island (3) Katchall (4) Little Andaman Which of the following code depicts the islands located in north to south order? (a) 1-2-3-4 (b) 2-1-4-3 (c) 2-1-3-4 (d) 1-2-4-3

Answer: (b) Explanation: The correct order of the islands from North to South is Narcondam-Barren-Little Andaman- Katchall.

7. Consider the following statements regarding Pitts India Act, 1784: (1) The Act provided for the complete transfer of control from the hands of East India Company to the British Government. (2) The Governor General's Council was reduced to five members, one of whom was Commander- in-chief of the King’s army in India. Which of the statements given above is/are correct? (a) 1 only (b) 2 only (c) Both 1 and 2 (d) Neither 1 nor 2

Answer: (d) Explanation: The Pitt’s India Act, 1784 also called the East India Company Act, 1784 was passed by the British Parliament to correct the defects of the Regulating Act of 1773. This act resulted in dual control of British possessions in India by the British government and the Company with the final authority resting with the government. This act continued in effect till 1858.  For political matters, Board of Control was created and for commercial affairs, the Court of Directors was appointed.  The Board of Control took care of civil and military affairs. It comprised of 6 people: o Secretary of State (Board President) o Chancellor of the Exchequer o Four Privy Councillors  In this dual system of control, the company was represented by the Court of Directors and the British government by the Board of Control.  The act mandated that all civil and military officers disclose their property in India and Britain within two months of their joining.  The Governor-General’s council’s strength was reduced to three members. One of the three would be the Commander-in-Chief of the British Crown’s army in India.  The Governor-General was given the right of veto.  The Presidencies of Madras and Bombay became subordinate to the Bengal Presidency. In effect, Calcutta became the capital of the British possessions in India.

8. Consider the following banks: (1) State Bank of Bikaner and Jaipur (SBBJ). (2) State Bank of Mysore (SBM) 3

(3) State Bank of Travancore (SBT) (4) State Bank of Hyderabad (SBH) (5) State Bank of Patiala (SBP) (6) Bharatiya Mahila Bank Which of the above banks have been merged with the State Bank of India? (a) 1, 2, 4 and 5 only (b) 1, 2, 3, 4 and 5 only (c) 2, 3, 4 and 6 only (d) 1, 2, 3, 4, 5 and 6

Answer: (d) Explanation: The boards of State Bank of Bikaner and Jaipur (SBBJ), State Bank of Mysore (SBM), State Bank of Travancore (SBT), the unlisted State Bank of Hyderabad (SBH), State Bank of Patiala (SBP) and Bharatiya Mahila Bank approved the scheme of merger with State Bank of India.

9. Recently Odisha’s Sualgiri and Swalgiri communities have been notified as Scheduled Castes. In this regard, consider the following statements: (1) The Constitution of India envisages that any inclusion in or exclusion from the list of Scheduled Castes in a state/UT can be done only through an Act of Parliament. (2) According to the Constitution, the Governor of the State concerned is required to approve the inclusion of a caste from the State in the ‘Scheduled’ list. (3) If the National Commission of SC’s requests the Central Government to include a caste in the 'Scheduled' list, then it is binding upon the Central government to do so. Which of the statements given above is/are correct? (a) 1 only (b) 2 and 3 only (c) 1 and 3 only (d) None of the above

Answer: (a) Explanation: Criteria for inclusion in SC list: Criteria for inclusion in SC list:  Scheduled Castes (SCs):- Extreme social, educational and economic backwardness arising out of traditional practice of untouchability. Procedure:  The Constitution of India empowers the Parliament of India to modify the list of notified scheduled castes, which upon receiving the assent of the President of India can be notified as Scheduled Castes.  Under the provision of Article 341, list of SCs in relation to a states/UT is to be issued by a notified Order of the President after consulting concerned State Government.  However, the clause (2) of Article 341 envisages that, any subsequent inclusion in or exclusion from the list of Scheduled Castes can be effected only through an Act of Parliament.  Further, Government has laid down Modalities in June, 1999, as amended in June, 2002 for processing of modifications in the lists of SCs and STs.  The Modalities envisage that only such proposals of the concerned State Governments / Administrations, which have been agreed to by the Registrar General of India (RGI) and National Commission for Scheduled Castes (NCSC), in the case of SCs, are further processed in accordance with provisions of clause (2) of Articles 341 and 342 respectively.

10. Consider the following statements regarding the tropical evergreen and semi evergreen forests: (1) These forests are generally located in areas with annual rainfall more than 250 cm and average annual temperature above 22 °C (2) They are found along the western slopes of Western Ghats and in the hills of north-eastern States (3) The timber from these forests is hard and of low economic value Which of the statements given above is/are INCORRECT? (a) 1 and 3 only (b) 1 and 2 only 4

(c) 3 only (d) 1, 2 and 3

Answer: (c) Explanation: Tropical Evergreen Forests Conditions for growth: Tropical Evergreen and Semi Evergreen Forests are found mainly in the areas where the annual rainfall is more than 250 cm, with a short dry season. The average annual temperature should be above 22 °C. Characteristics: Lofty, very dense, multi-layered forest with mesophytic evergreen, 45m or more in height, with large number of species, numerous epiphytes, and few climbers; Due to dense growth of trees, the sunlight cannot reach the ground. Thus, the undergrowth mainly consists of canes, bamboos, ferns, climbers, etc. Location: The true evergreen forests are mostly found along the western slopes of Western Ghats, in the hills of north-eastern states and in the Andaman and Nicobar Islands. Trees: Important trees of these forests are rosewood, ebony, mahogany, rubber, cinchona, bamboo, , palms, canes, lianas, etc. Utility: Not commercially exploitable. However, the timber from the tropical evergreen and semi- evergreen a forest is hard, durable, fine-grained and of high economic value.

11. The Ministry of Earth Sciences (MoES) has inaugurated High Performance Computer System (HPC) (Supercomputer Mihir) at the National Centre for Medium Range Weather Forecasting (NCMRWF) located at: (a) Mumbai (b) Bangalore (c) Ahmedabad (d) NOIDA

Answer: (d) Explanation: Ministry of Earth Sciences (MoES) has inaugurated high performance computer system (HPC) or supercomputer Mihir (meaning sun) at National Centre for Medium Range Weather Forecasting (NCMRWF), Noida. The HCP will provide facility for improving weather/climate forecasts. It is country’s largest HPC facility in terms of peak capacity and performance. It also propelled India’s ranking to Top 30 in the list of HPC facilities in world

12. According to the National Commission on Agriculture there are 3 types of droughts. Which of the following is NOT one of them? (a) Meteorological (b) Agricultural (c) Hydrological (d) Ecological

Answer: (d) Explanation: Classification of droughts The National Commission on Agriculture in India classified three types of drought: meteorological, agricultural and hydrological.  Meteorological drought is defined as a situation when there is significant decrease from normal precipitation over an area (i.e. more than 10 %).  Hydrological drought results from prolonged meteorological drought resulting in depletion of surface and sub-surface water resources.  Agricultural drought is a situation when soil moisture and rainfall are inadequate to support healthy crop growth.

13. Consider the following Commissions of pre-1947 India and their field of study:

Commission Field 1. Sadler Commission : Education 5

2. Strachey Commission : Famine 3. Fraser Commission : Agriculture

Which of the above pairs is/are correctly matched? (a) 1 only (b) 1 and 2 only (c) 2 and 3 only (d) 1, 2 and 3

Answer: (b) Explanation: Lord Curzon had first tried to introduce some administrative reform of the university education by instituting a University Education Commission in 1902. At the time of the Government of India Resolution in 1913 there were only five universities in India and the number of colleges was beyond the control of the various universities within their territorial limits. As a result, different administrative problems piled up in that period. Therefore, it became a necessity to reform the Indian Universities also. All these circumstances led to the formation of the Second University Commission. i.e., Calcutta University Commission in 1917 and report on the problems of university education. The Commission is also known as the Sadler Commission after the name of its chairman Dr. Michael E. Sadler. Due to the suggestions of this Commission a number of new universities were opened in the country. Of these, the universities at Patna, Osmania, Aligarh, Dacca, Lucknow, Delhi, Agra, Nagpur, Hyderabad and Annamalai may be mentioned. The number increased upto 30 within 1930.

The recurring famines of 19th century were the inevitable consequences of the British policies and exposed the real character of the British administration for Indian peasantry. Amongst all such tragedies, a terrible bout of famine struck parts of South India in 1877. In May 1878, Lord Lytton appointed a Famine Commission with Sir Richard Strachey (1817-1908) as President and Sir Charles A. Elliott (1835-1911) as Secretary. The Commission was charged to investigate the impact of the famine on vital statistics, to suggest the character of future relief programmes, to outline criteria for complimentary relief, to clarify the Government's role in the supply and distribution of food and to project the expected benefit of railway and irrigation projects. The Commission recommended: 1. A famine code should be formulated. 2. Irrigation facilities should be developed. 3. Collection of land revenue should be suspended immediately during famines and land revenue should be remitted. 4. Data should be collected about the conditions of Indian peasantry and agriculture. 5. A famine fund should be set up.

In 1902-03, a Police Commission was established for the Police reforms under Sir Andrew Frazer by Lord Curzon. It recommended the appointment of Indians at officer level in the police. Indians could rise only to the ranks of Inspector of Police, the senior N.C.O. position. However, they were not part of Indian Imperial Police.

14. Consider the following statements: (1) Article 35A of the Indian Constitution empowers the Jammu & Kashmir legislature to define “permanent residents” of the State and provide special rights and privileges to those permanent residents. (2) Article 1 and Article 370 are the only articles of Indian Constitution which apply of their own force to the State of Jammu and Kashmir. Which of the statements given above is/are correct? (a) 1 only (b) 2 only (c) Both 1 and 2 (d) Neither 1 nor 2

Answer: (c) Explanation: 6

 The Jammu &Kashmir Constitution, which was adopted on November 17, 1956, defined a Permanent Resident as a person who was a state subject on May 14, 1954, or who has been a resident of the State for 10 years, and has “lawfully acquired immovable property in the state”.  Article 35A is a provision in the Constitution that empowers the Jammu & Kashmir legislature to define “permanent residents” of the state and provide special rights and privileges to those permanent residents.  Section 6 of the Jammu and Kashmir Constitution restricts the basic right of women to marry a man of their choice by not giving the heirs any right to property if the woman marries a man not holding the Permanent Resident Certificate.  Earlier, a 2002 order by the Jammu &Kashmir High Court in the State of Jammu and Kashmir vs. Dr. Sushila Sawhney and Others had said the daughter of a permanent resident marrying a person outside the State would not lose the status of permanent resident of Jammu & Kashmir.

Article 1 and Article 370 are the only articles of Indian Constitution which apply of their own force to the State of Jammu and Kashmir. All other articles are extended to the State by notification of the President after consultation with the Jammu & Kashmir State legislative assembly (Art. 370)

15. With reference to the “Bharat QR Code”, consider the following statements: (1) Bharat QR code is an interoperable payment acceptance solution that supports Visa, Master Card, Amex and RuPay cards and BHIM-UPI. (2) It is a P2M (Person to Merchant) Mobile payment solution. (3) It makes a PoS (Point of Sale) swiping machine redundant. Which of the statements given above is/are correct? (a) 1 only (b) 2 and 3 only (c) 1 and 2 only (d) 1, 2 and 3

Answer: (d) Explanation: Bharat QR code is an interoperable payment acceptance solution that supports Visa, MasterCard. Amex and RuPay cards & BHIM-UPI for wider acceptance. Bharat QR code enables rapid rollout of digital payments acceptance infrastructure throughout the country, as it does not involve any upfront investment in Point of Sale (PoS) machine. Bharat QR is P2M (Person to Merchant) Mobile payment solution. To facilitate massive rollout in a short span of time, Bharat QR code based payment solution is introduced with following advantages: 1. Bharat QR code does not require any upfront expenditure. 2. Bharat QR code is a single unified QR code capable of accepting payments from Visa, MasterCard, RuPay Cards for wider acceptance. 3. Customer can easily make payments through Bharat QR code and does not require to carry physical Debit or Credit card. 4. The risk of data theft or security issues through tampered or cyber-compromised point of sale devices is minimised. 5. Bharat QR code supports dynamic QR codes, which may be printed on electricity bills, gas bills and other utility bills to make payments to the respective vendors. 6. Merchants accepting the payment through Bharat QR Code, receives the amount directly in their Bank account. Bharat QR code is different from UPI QR code which is used for P2P (person to person) transactions as well.

16. The maximum number of recipients of India’s top Civilian Awards (Bharat Ratna and Padma Awards) who are living abroad are from which of the following countries?

7

(a) USA (b) UK (c) Russia (d) Japan

Answer: (a) Explanation:

17. With reference to the ‘waterfalls in India’, consider the following statements: (1) Duduma falls are situated in Madhya Pradesh on the river Narmada. (2) Shivasamudram falls are in Tamil Nadu on the river Kaveri. Which of the statements given above is/are incorrect? (a) 1 only (b) 2 only (c) Both 1 and 2 (d) Neither 1 nor 2

Answer: (c) Explanation: 1. Dhuandhar falls are situated in Jabalpur district of Madhya Pradesh on the River Narmada 2. Shivasamudram falls are in Karnataka on the river Kaveri 3. Dudama falls are situated 90 kms from Koraput district of Odisha and are situated on river Muchkund

18. Consider the following pairs:

1. Warren : Established the Calcutta Madarsa in 1781 for the teaching of Muslim Hastings Law. 2. Sir Charles : His report is called the “Magna Carta of English Education in India”. Wood 3. Jonathan : Started a Sanskrit College in Varanasi in 1791 for the study of Hindu Duncan philosophy and laws.

Which of the above pairs is/are correctly matched? (a) 1 only (b) 1 and 2 only (c) 2 and 3 only (d) 1, 2 and 3

Answer: (d) Explanation:  To understand the local customs and laws, Warren Hastings established the Calcutta Madarsa in 1781 for the teaching of Muslim law.

8

 Sir Charles Wood was the President of the Board of Control of the company in 1854 when he sent a despatch to the then Governor-General of India, Lord Dalhousie. This is called the ‘Magna Carta of English education in India.’  In 1791, a Sanskrit College was started in Varanasi by Jonathan Duncan for the study of Hindu philosophy and laws.

19. With reference to deficiency disorders, consider the following statements: (1) Weak muscles and very little energy to work may be symptoms of Beriberi which happens due to deficiency of Vitamin B12. (2) The absence of zinc is associated with anaemia, and impaired cognitive and motor functions. Which of the statements given above is/are correct? (a) 1 only (b) 2 only (c) Both 1 and 2 (d) Neither 1 nor 2

Answer: (b) Explanation: 1. Deficiency of Vitamin B1 causes Beriberi 2. Deficiency of Vitamin B9 & Vitamin B12 causes Megaloblasticanemia Zinc: Found in a range of foodstuffs including liver, eggs, nuts, cereals and seafood. The absence of zinc is associated with a number of conditions including, short stature, anaemia, impaired healing of wounds, poor gonadal function, and impaired cognitive and motor function. It can also lead to appetite disorders, as well as contributing to the increased severity and incidence of diarrhoea and pneumonia. The most important effect of zinc deficiency is its impact on children’s resistance to infectious diseases including the risk of infection, the recurrence of infections and the severity of infection. This is well document in the case of diarrhoea. Zinc nutrition is therefore an important determinant of mortality in children.

20. Which of the following Ramsar sites is NOT in the Indian state of Punjab? (a) Rudrasagar (b) Harike Lake (c) Kanji Lake (d) Ropar Lake

Answer: (a) Explanation Rudrasagar Lake is in Tripura.

21. Consider the following statements with respect to backward classes: (1) Article 330 of the Indian Constitution allows the President to appoint a Commission to investigate the conditions of socially and economically backward classes and table the report in the Parliament. (2) So far, 11 States/UTs have carried out sub-categorisation of OBCs and the Central List of OBCs has been accordingly amended. Which of the statements given above is/are correct? (a) 1 only (b) 2 only (c) Both 1 and 2 (d) Neither 1 nor 2

Answer: (d) Explanation:  The President has appointed a five-member Commission headed by Delhi High Court's former Chief Justice G Rohini to examine the sub-categorisation of Other Backward Classes (OBCs) under

9

Article 340 of the Constitution. It allows the President to appoint a Commission to investigate the condition of socially and economically backward classes and table the report in the Parliament.  The Supreme Court of India in Indra Sawhney vs. Union of India case (1992) had observed that there is no constitutional or legal bar on states for categorizing OBCs as backward or more backward. It had also observed that it is not impermissible in law if state chooses to do sub- categorisation.  So far, 11 States/UTs viz. Karnataka, Haryana, Andhra Pradesh, Jharkhand, Puducherry, Telangana, West Bengal, Bihar, Maharashtra, Tamil Nadu and Jammu region of Jammu & Kashmir have carried out sub-categorization of OBCs. However, there was no sub categorisation in central list of OBCs.

22. India’s first dedicated space observatory, ASTROSAT has captured the rare phenomenon of a “Blue Straggler” star in 2017. What is a “Blue Straggler” star? (a) It is a star which feeds off its companion star by sucking out its mass and energy, causing its eventual death (b) It is a highly magnetized, rotating neutron star that emits a beam of electromagnetic radiation (c) It is a type of compact exotic star, where extremely high temperature and pressure has forced nuclear particles to form a continuous state of matter that consists primarily of free quarks (d) It is a star with distinctly unusual metal abundances, at least in their surface layers

Answer: (a) Explanation: India’s first dedicated space observatory, Astrosat, has captured the rare phenomenon of a small, 6-billion- year-old “vampire” star “preying” on a bigger celestial body. Scientists say the smaller star, also called a “blue straggler”, feeds off its companion star by sucking out its mass and energy, causing its eventual death. The small star becomes bigger, hotter and bluer, which gives it the appearance of being young, while the ageing companion burns out and collapses to a stellar remnant. ASTROSAT is India’s first dedicated multi wavelength space observatory. This scientific satellite mission endeavours for a more detailed understanding of our universe. One of the unique features of ASTROSAT mission is that enables the simultaneous multi-wavelength observations of various astronomical objects with a single satellite. It was launched in 2015 by ISRO. Option (b) is Pulsar Star Option (c) is Quark Star Option (d) is Chemically Peculiar Star

23. With reference to the BBIN Agreement, consider the following statements: (1) The Bangladesh-Bhutan-India-Nepal (BBIN) Motor Vehicle Agreement (MVA) signed in June 2015 has been ratified by Bangladesh, India and Nepal. (2) Bhutan has opted out of the agreement due to Chinese pressure following the Doklam issue. Which of the statements given above is/are correct? (a) 1 only (b) 2 only (c) Both 1 and 2 (d) Neither 1 nor 2

Answer: (a) Explanation:  Bangladesh, India and Nepal have agreed on the text of the operating procedures for passenger vehicle movement in the sub-region under the Bangladesh-Bhutan-India-Nepal (BBIN) Motor Vehicles Agreement (MVA) signed in June 2015, and will soon complete the internal approval processes for signing of the passenger protocol. The participating countries have also agreed to conduct more trial runs for cargo vehicles under the agreement. The landmark MVA was signed by Transport Ministers of the BBIN countries in Thimphu, Bhutan on 15 June 2015.  The Bhutanese Council believes that ratifying the agreement, which aims to facilitate seamless cross- border movement of both cargo and passenger vehicles, will overwhelm Bhutan, the smallest country in the grouping. The opposition also argues that it will affect the livelihoods of Bhutanese transport

10

operators. However, the main reason behind Bhutan’s hesitance to sign the pact now is because of environmental concerns raised by a section of the country’s citizens.

24. Through how many states, River Krishna flows before falling into the Bay of Bengal? (a) 2 (b) 3 (c) 4 (d) 5

Answer: (c) Explanation: Krishna River flows through 4 states of Maharashtra, Karnataka, Telangana, Andhra Pradesh.

25. With reference to the Harappan seals, consider the following statements: (1) Horse was the animal most commonly represented on the Harappan seals. (2) Cow was the animal which was not represented in any seals and terracotta art of the Harappan culture. Which of the statements given above is/are correct? (a) 1 only (b) 2 only (c) Both 1 and 2 (d) Neither 1 nor 2

Answer: (b) Explanation:  Archaeologists have discovered thousands of seals, usually made of steatite, and occasionally of agate (a variety of silica, chiefly characterised by its fineness of grain and brightness of colour), chert (a hard, dark, opaque rock composed of silica), copper, faience (glazed ceramic ware) and terracotta, with beautiful figures of animals, as unicorn bull, rhinoceros, tiger, elephant, bison, buffalo, etc.  Unicorn (bull) was the animal most commonly represented on the seals. Among the numerous seals found in Harappa there is none which represent a horse, while other animals like the bull, buffalo, and goat are represented. The realistic rendering of these animals in various moods is remarkable.  The purpose of producing seals was mainly commercial. It appears that the seals were also used as amulets, perhaps as modern-day identity cards.  Every seal is engraved in a pictographic script, which is yet to be deciphered. Some seals have also been found in gold and ivory.

26. Which of the following was NOT a member of the Drafting Committee of the Constitution? (a) G.V. Mavalankar (b) Mohammad Saadulla (c) D.P. Khaitan (d) B.L. Mitter

Answer: (a) Explanation: On 29th August 1947, the Constituent Assembly through a resolution appointed a Drafting Committee to: “..scrutinise the draft of the text of the Constitution of India prepared by Constitutional Adviser, giving effect to the decisions already taken in the Assembly and including all matters which are ancillary thereto or which have to be provided in such a Constitution, and to submit to the Assembly for consideration the text of the draft constitution as revised by the committee” The Drafting Committee had seven members: Alladi Krishnaswami Ayyar, N. Gopalaswami; B.R. Ambedkar, K.M. Munshi, Mohammad Saadulla, B.L. Mitter and D.P. Khaitan. At its first meeting on 30th August 1947, the Drafting Committee elected B.R. Ambedkar as its Chairman.

27. Consider the following types of Carnatic Music:

11

Type of Carnatic Music Meaning 1. Tanam-Pallavi : Performed at the beginning of a concert, a completely composed piece. 2. Padam : Slower tempoed love songs referring to the human yearning for the adored God head. 3. Javalis : Elaborate rhythmic and melodic variation in unmeasured sense.

Which of the pairs given above is/are correctly matched? (a) 1 only (b) 1 and 2 only (c) 2 only (d) 2 and 3 only

Answer: (c) Explanation: Ragam  Tanam- Pallavi - Elaborate rhythmic and melodic variation in unmeasured sense.  Kriti-Kirthanai - Most popular type which refers to devotional music laced with poetic beauty.  Varnam- Performed at the beginning of a concert ; a completely composed piece.  Padam - Slower tempoed love songs referring to the human yearning for the adored god head.  Javalis - Faster tempoed love songs with direct description of human love.  Tillana- Meaningful phrases are interspersed with variety of meaningless syllables.

28. Consider the following statements regarding the Annual Status of Education Report(ASER) and National Achievement Survey (NAS): (1) ASER is a household survey, whereas NAS is a School based effort (2) ASER is a pen and paper test administered to a group of students in school, whereas, NAS- reading and arithmetic assessments are administered one on one in an oral format. Which of the statements given above is/are correct? (a) 1 only (b) 2 only (c) Both 1 and 2 (d) Neither 1 nor 2

Answer: (a) Explanation: In News: ASER is a household survey. ASER reading and arithmetic assessments are administered one on one in an oral format. Children are tested at home. All children are given the same test, regardless of age or grade. NAS-Class V is a pen and paper test administered to a group of students in school. In most schools, children were tested in two out of the three subjects. The cover of the test booklet has instructions for students indicating how to record or modify their responses. http://img.asercentre.org/docs/Bottom%20Panel/Key%20Docs/nas_stdvvs_aser.pdf

29. With reference to the “Small Finance Banks”, consider the following statements: (1) Resident individuals/professionals with 10 years of experience in banking and finance, and companies and societies owned and controlled by residents will be eligible to set up small finance banks. (2) The minimum paid-up equity capital for small finance banks shall be Rs. 500 crore. Which of the statements given above is/are correct? (a) 1 only (b) 2 only (c) Both 1 and 2 (d) Neither 1 nor 2

Answer: (a) Explanation: 12

Small Finance Banks (SFBs)  They are niche banks that focus and serve the needs of a certain demographic segment of the population.  The objectives of setting up of small finance banks will be to further financial inclusion by (1) the provision of savings vehicles (2) supply of credit to small business units; small and marginal farmers; micro and small industries; and other unorganised sector entities, through high technology- low cost operations.  SFBs was recommended by the Nachiket Mor committee on financial inclusion. Scope of activities of SFBs  The small finance banks shall primarily undertake basic banking activities of acceptance of deposits and lending to unserved and underserved sections including small business units, small and marginal farmers, micro and small industries and unorganised sector entities.  There will not be any restriction in the area of operations of small finance banks. Criteria for setting up SFBs  Resident individuals/professions with 10 years of experience in finance, Non-Banking Financial Companies (NBFCs), micro finance companies, local area banks are eligible to set up SFBs.  The minimum paid-up equity capital for small finance banks shall be Rs. 100 crore.  The promoter’s minimum initial contribution to the paid-up equity capital of such small finance bank shall at least be 40 per cent and gradually brought down to 26 per cent within 12 years from the date of commencement of business of the bank.  The foreign shareholding in the small finance bank would be as per the Foreign Direct Investment (FDI) policy for private sector banks as amended from time to time.  The small finance banks will be required to extend 75 per cent of its Adjusted Net Bank Credit (ANBC) to the sectors eligible for classification as priority sector lending (PSL) by the Reserve Bank.  SFBs have to maintain Cash Reserve Ratio (CRR) and Statutory Liquidity Ratio (SLR) as per RBI norms.  At least 50 per cent of its loan portfolio should constitute loans and advances of up to Rs. 25 lakh. What can Small Finance Banks do?  Sell forex to customers.  Sell mutual funds, insurance and pensions.  Can convert into a full-fledged bank. What Small Finance Banks can’t do?  Extend large loans.  Cannot float subsidiaries and deal in sophisticated products. Challenges to Small Finance Banks  Have to compete with existing public sector banks and RRBs.  Micro Finance Institution (MFI)/NBFC are specialised in micro lending operations with limited exposure to banking operations; that means they have to hire, train talent from the banking industry.  The cost of deposit mobilisation will be higher for these banks as they cover rural and underserved segment

30. Consider the following statements: (a) Accreditation Master Agreement (AMA) was signed between Green Climate Fund and NABARD. (b) India has set a target for achieving 40% power generation from non-fossil based sources by 2030 under its Nationally Determined Contributions. Which of the statements given above is/are correct? (a) 1 only (b) 2 only (c) Both 1 and 2 (d) Neither 1 nor 2

Answer: (c) Explanation: Green Climate Fund [GCF], set up by UNFCCC in 2010, for supporting developing countries to mitigate adverse impacts of climate change has decided to operate in India through NABARD. NABARD has been accredited by GCF as the first entity for sourcing financial resources from GCF for India. GCF resources

13

can be accessed by different countries only through Accredited entities like NABARD, which will be supporting public and private sector initiatives in India. India aims to provide leadership to the world in respect of renewable energy segment and a target for achieving 40% power generation from non-fossil based sources by 2030 has been set to achieve carbon emission intensity reduction target under its Nationally Determined Contributions submitted under Paris Climate Summit 2015.

31. With reference to the “UNESCO Natural World Heritage Sties”, consider the following statements: (1) Gujarat has no site in the list. (2) Assam boasts of two sites in this list. Which of the statements given above is/are correct? (a) 1 only (b) 2 only (c) Both 1 and 2 (d) Neither 1 nor 2

Answer: (c) Explanation: Natural World Heritage Sites in India

Sl. Name of WH Site State No. Location 1 Great Himalayan National Park Himachal Pradesh

Conservation Area

2 Western Ghats Maharashtra, Goa, Karnataka, Tamil Nadu and Kerala 3 Nanda Devi and Valley of Flowers Uttarakhand National Parks

4 Sundarbans National Park West Bengal

5 Kaziranga National Park Assam

6 Keoladeo National Park Rajasthan

7 Manas Wildlife Sanctuary Assam

32. Consider the following statements regarding the Rowlatt Act: (1) It was officially known as the Anarchical and Revolutionary Crimes Act, 1919. (2) It also authorised the government to arrest people for up to 6 months without trial. Which of the statements given above is/are correct? (a) 1 only (b) 2 only 14

(c) Both 1 and 2 (d) Neither 1 nor 2

Answer: (a) Explanation:  Passed in March 1919 by the Imperial Legislative Council, it was officially known as the Anarchical and Revolutionary Crimes Act, 1919.  This act authorised the British government to arrest anybody suspected of terrorist activities.  It also authorised the government to detain such people arrested for up to 2 years without trial.  It empowered the police to search a place without a warrant and placed severe restrictions on the freedom of the press  The act was passed as per recommendations of the Rowlatt Committee chaired by a judge, Sir Sidney Rowlatt.  Punjab was put under martial law which meant that it became unlawful for more than 4 people to assemble at a place.  The Lieutenant-Governor of Punjab at that time was Michael O’Dwyer. Lord Chelmsford was India’s Viceroy.

33. Consider the following statements regarding the Office of Profit: (1) It is defined in the Constitution under Article 102(1)(a). (2) In the Indian Constitution Article 102 and Article 191 deal with the disqualification of Members of Parliament and the Members of the State Legislature respectively. Which of the statements given above is/are correct? (a) 1 only (b) 2 only (c) Both 1 and 2 (d) Neither 1 nor 2

Answer: (b) Explanation: What is an office of profit? It is a position in the government which cannot be held by an MLA or an MP. The post can yield salaries, perquisites and other benefits. The origin of this term can be found in the English Act of Settlement, 1701. Under this law, "no person who has an office or place of profit under the King, or receives a pension from the Crown, shall be capable of serving as a member of the House of Commons." This was instituted so that there wouldn't be any undue influence from the royal household in administrative affairs. Why should an MLA or an MP not hold an office of profit? According to Articles 102(1)(a) and 191(1)(a) of the Constitution, an MP or MLA is barred from holding an office of profit as it can put them in a position to gain a financial benefit. "A person shall be disqualified for being chosen as, and for being, a member of either House of Parliament, (a) if he holds any office of profit under the Government of India or the Government of any State, other than an office declared by Parliament by law not to disqualify its holder," says the law. Under the Representation of People Act too, holding an office of profit is grounds for disqualification.

34. Consider the following groups into which India was recently admitted: (1) Missile Technology Control Regime (MTCR) (2) Australia Group (3) Wassenaar Arrangement Which of the code given below correctly depicts the order of India joining these groups from first to the last? (a) 1-2-3 (b) 3-2-1 (c) 2-1-3 (d) 1-3-2

Answer: (d) Explanation:

15

(1) Missile Technology Control Regime (MTCR) – India admitted in June 2016. (2) Australia Group-India admitted in January 2018. (3) Wassenaar Arrangement – India admitted as “Participating State” in December 2017.

35. With reference to “terrestrial sequestration”, which of the following has the highest CO2 sequestration rate? (a) Cropland (b) Grassland (c) Forest (d) Wetland

Answer: (d) Explanation:

Range of CO sequestration rate Biome 2 (metric tons/acre/year) Cropland 0.2 - 0.6 Forest 0.05 - 3.9 Grassland 0.12 – 1.0 Swamp/Floodplain/Wetland 2.23 – 3.71

36. Consider the following statements regarding Insolvency and Bankruptcy Board of India (IBBI): (1) The Centre has constituted a four-member Board. (2) The main activity of the Board would be to regulate the functioning of insolvency professionals, insolvency professional agencies and information utilities under the Insolvency and Bankruptcy Code 2016. (3) It is to be directly administered by the Ministry of Corporate Affairs. Which of the statements given above is/are incorrect? (a) 1 only (b) 1 and 2 only (c) 3 only (d) None of the above

Answer: (d) Explanation:  The Central Government had constituted a four-member Insolvency and Bankruptcy Board of India (IBBI) under the Chairmanship of MS Sahoo in October 2016.  While the Centre has for now set up the IBBI with four members, going forward this will be expanded to 10 (including the Chairman) as provisioned in Insolvency and Bankruptcy code.  Three whole-time members and two other members are to be appointed, which will take the overall tally to 10 in the coming days.  The main activity of IBBI would be to regulate the functioning of insolvency professionals, insolvency professional agencies and information utilities under the Insolvency and Bankruptcy Code 2016.  It is under Ministry of Corporate Affairs.

37. Who among the following Europeans, were the last to come to Pre-independence India and establish their trading factories? (a) The Dutch (b) The French (c) The Portuguese (d) The English

Answer: (b) Explanation: Timeline of the Europeans’ advent into Pre-independence India is: 16

 Portuguese – 1498 AD (1st factory was established in Cochin in 1503AD)  English – 1600 AD (1st Factory was established in Surat in 1608 AD)  Dutch – 1602 AD (1st Factory was established in Masulipattanam in 1605 AD)  French – 1664 AD (1st Factory was established in Surat in 1668 AD)

38. Postal Life Insurance was in news with reference to the Sampoorna Bima Gram (SBG) Yojana. In this context consider the following statements: (1) Postal Life Insurance (PLI) was introduced in 1884 and is the oldest life insurance scheme in India. (2) Under the Sampoorna Bima Gram Yojana all households in at least ten villages (having minimum 1000 households per village)in a revenue district are to be covered compulsorily with a Rural PLI. Which of the statements given above is/are correct? (a) 1 only (b) 2 only (c) Both 1 and 2 (d) Neither 1 nor 2

Answer: (a) Explanation:  The Union Government has launched the Sampoorna Bima Gram (SBG) Yojana and an initiative for expansion of clientele base of Postal Life Insurance so as to enable Rural people to get affordable life insurance services.  The idea is to use postal networks to provide affordable life insurance services to people living in rural areas of the country. All villages under the Saansad Adarsh Gram Yojana will be brought under its ambit.  Under Sampoorna Bima Gram (SBG) Yojana, at least one village (having a minimum of 100 households) will be identified in each of the revenue districts of the country, wherein endeavour will be made to cover all households of that identified village with a minimum of one RPLI (Rural Postal Life Insurance) policy each. PLI:  It was introduced in 1884 and is one of the oldest life insurance schemes for benefit of Government and semi-Government employees.  Rural Postal Life Insurance (RPLI), introduced on March 24, 1995 on recommendations of Malhotra Committee, provides insurance cover to people residing in rural areas, especially weaker sections and women living in rural areas.

39. Consider the following statements regarding the Missile Technology Control Regime (MTCR): (1) Established in 1987, it is a 35 member multilateral control regime to which India was admitted in June 2016. (2) While China is not a member, India is the only one of the four unrecognised nuclear powers to be a member. Which of the statements given above is/are correct? (a) 1 only (b) 2 only (c) Both 1 and 2 (d) Neither 1 nor 2

Answer: (c) Explanation:  It is a multilateral, consensus – based grouping of 35 member countries who are voluntarily committed to the non-proliferation of missiles capable of carrying chemical, biological and nuclear weapons of mass destruction (WMDs).  MTCR aims at restricting the proliferation of missiles, complete rocket systems, unmanned air vehicles and related technology for those systems capable of carrying a 500 kg payload for at least 300 km, as well as systems intended for the delivery of weapons of mass destruction (WMD).

17

 It was established in April 1987 by G-7 countries – USA, UK, France, Germany, Canada, Italy, and Japan, to check the spread of unmanned delivery systems capable of carrying nuclear weapons of above 500kg for more than 300km. In 1992, it was extended for all types of weapons of mass destruction.  Now, it has 35 full members including India and 4 “non-adherent members” – Israel, Macedonia, Romania, Slovakia.  China is not a member of this regime but it had verbally pledged to adhere to its original guidelines but not to the subsequent additions.

40. Consider the following statements: (1) The Central Government has released India’s second Wildlife Action Plan 2017-2031 after first such plan of 2002. (2) The Central Government has launched “Secure Himalaya” scheme to increase the water content of all Himalayan rivers by securing the glaciers. Which of the statements given above is/are correct? (a) 1 only (b) 2 only (c) Both 1 and 2 (d) Neither 1 nor 2

Answer: (d) Explanation:  The central government has released India’s third wildlife action plan, to chalk out India’s strategy for wildlife conservation in the next fifteen years.  The first wildlife action plan was unveiled in 1983 and the second in 2002.  This plan was drafted by a 12-member committee chaired by JC Kala, a former secretary to the ministry.  The plan adopts a “landscape approach” to the conservation of all wildlife that has an ecological value.  The aim of the scheme "Secure Himalaya" is to protect Snow leopards by saving their habitats and to improve the ecology of life of the Himalayan ranges and hill communities- all among the worst victims of climate change.  The Secure Himalaya project, launched by the Union Environment Ministry in collaboration with United Nations Development Program (UNDP).  Himachal Pradesh, Jammu and Kashmir, Uttarakhand and Sikkim will be covered.

41. The Tropic of Cancer passes through which of the following States? (1) Rajasthan (2) Jharkhand (3) Manipur (4) Arunachal Pradesh (5) West Bengal (6) Tripura Select the correct answer using the code given below: (a) 1, 2, 3 and 5 only (b) 1, 2, 4, 5 and 6 only (c) 2, 3, 5 and 6 only (d) 1, 2, 5 and 6 only

Answer: (d) Explanation: The Tropic of Cancer passes through following States 1. Gujarat 2. Rajasthan 3. Madhya Pradesh 4. Chhattisgarh 5. Jharkhand 6. West Bengal 18

7. Tripura 8. Mizoram

42. Consider the following pairs:

Organisation Founder 1. Prarthana Samaj : Devendranath Tagore 2. Tatvabodhini Sabha : Dr. Atmaram Pandurang 3. Deva Samaj : Shivanarayan Agnihotri

Which of the above pairs is/are correctly matched? (a) 1 only (b) 1 and 2 only (c) 2 and 3 only (d) 3 only

Answer: (d) Explanation:

Organisation Founder 1. Prarthana Samaj : Dr. Atmaram Pandurang 2. Tatvabodhini Sabha : Devendranath Tagore 3. Deva Samaj : Shivanarayan Agnihotri

43. With reference to the “Fisheries Sector in India”, consider the following statements: (1) Recently, India became the second largest fish producing country in the world. (2) Marine fisheries contribute to around 65% of the total fish catch/production, whereas the rest comes from inland fisheries. Which of the statements given above is/are correct? (a) 1 only (b) 2 only (c) Both 1 and 2 (d) Neither 1 nor 2

Answer: (a) Explanation:  Recently, India became second largest fish producing country in the world.  There are two branches of fishery sector namely Inland Fisheries and Marine Fisheries. The total fish production has nearly 65% contribution from the inland sector and rest from marine fishing.  Fish and fish products accounts for around 10% of the total exports of the country and nearly 20% of the agricultural exports.  Constituting about 6.3% of the global fish production, the sector contributes to 1.1% of the GDP and 5.15% of the agricultural GDP.

44. With reference to the “New Metro Rail Policy”, consider the following statements: (1) The policy makes private participation mandatory for availing central assistance for new metro projects. (2) New metro projects will be approved, on the basis of economic internal rate of return of 14%. Which of the statements given above is/are correct? (a) 1 only (b) 2 only (c) Both 1 and 2 (d) Neither 1 nor 2

Answer: (c) Explanation: Key points of the New Metro Rail Policy: 19

The policy gives a big boost to private players by making private participation mandatory for all the three funding options – be it a public-private partnership (PPP) model with central assistance under the Viability Gap Funding scheme of the Finance Ministry, a grant from the Centre under which 10% of the project cost would be given as a lump sum, or a 50:50 equity sharing model between the Central and State governments. Private participation “either for complete provision of metro rail or for some unbundled components” such as automatic fare collection will form an essential requirement for all metro rail projects seeking central financial assistance. The policy also seeks to ensure that metro projects are initiated for sound reasons. Every proposal for Metro Rail should necessarily include proposals for feeder systems that help to enlarge the catchment area of each metro station to at least 5 km. It also stipulates rigorous project evaluation by a third party. The policy also makes it mandatory for state governments to set up a unified metropolitan transport authority. This would be a statutory body entrusted with preparing a comprehensive mobility plan for the city. As per the policy, States are required to adopt innovative mechanisms such as ‘value capture financing’ and ‘betterment levy’ to mobilise resources for the project. States will also get a free hand in implementing the projects. In cases where States opt for central assistance of 10% of project cost, the Union government will not concern itself with project execution. Noting that metro projects should stop turning into white elephants, the policy stipulates an increase in rate of return from the current ‘Financial internal rate of return’ of 8% to ‘economic internal rate of return’ of 14%.

45. Regarding the Society for Worldwide Inter-Bank Financial Telecommunication (SWIFT), which was in the news, consider the following statements: (1) It is an international Financial Institution owned by the Central Banks which fosters international monetary and financial cooperation and serves as a bank for them. (2) It hosts the Secretariat of the Basel Committee on Banking Supervision and has played a central role in establishing the Basel III framework. Which of the statements given above is/are correct? (a) 1 only (b) 2 only (c) Both 1 and 2 (d) Neither 1 nor 2

Answer: (d) Explanation: Society for Worldwide Inter-Bank Financial Telecommunications (or SWIFT) is a member- owned cooperative society that provides safe and secure financial transactions for its members. Established in 1973, SWIFT uses a standardized proprietary communications platform to facilitate the transmission of information about financial transactions. Financial institutions securely exchange this information, including payment instructions, among themselves. SWIFT headquarters is in Belgium. SWIFT does not hold funds or manage accounts on behalf of customers. SWIFT neither holds funds on its own nor manages external client accounts.

46. Consider the following statements: (1) Fluorescence is a type of luminescence that is caused by the absorption of radiations (such as light or electrons) and continues for a noticeable time after these radiations have stopped. (2) Phosphorescence is the emission of light from an object as a result of bombardment by other kinds of electromagnetic radiation, such as x-rays or ultraviolet rays. Which of the statements given above is/are correct? (a) 1 only (b) 2 only (c) Both 1 and 2 (d) Neither 1 nor 2

Answer: (d) Explanation: (1) Phosphorescence is a type of luminescence that is caused by the absorption of radiations (such as light or electrons) and continues for a noticeable time after these radiations have stopped.

20

(2) Fluorescence is the emission of light from an object as a result of bombardment by other kinds of electromagnetic radiation, such as x-rays or ultraviolet rays.

47. With reference to the “Western Ghats”, consider the following statements: (1) The height of the Western Ghats increases from North to South. (2) Major ranges/hills of the region are Nallamala, Shevroy and Palkonda. Which of the statements given above is/are correct? (a) 1 only (b) 2 only (c) Both 1 and 2 (d) Neither 1 nor 2

Answer: (a) Explanation: Western Ghats extends from mouth of river Tapi to Kanyakumari. The average height of the hills is 1200mt. The height of the Western Ghats increases from North to South. Nilgiris, Annamalai and Cardamom hills are the major hills of the region. Nallamala, Shevroy, Javadi and Palkonda range are the hills of Eastern ghats, which are of lower height than Western Ghats.

48. Consider the following statements regarding the Indian Councils Act, 1861: (1) The Act introduced the system of elections in India. (2) It gave the Viceroy of India the power to promulgate ordinances, with the concurrence of the legislative council, during any emergency. Which of the statements given above is/are correct? (a) 1 only (b) 2 only (c) Both 1 and 2 (d) Neither 1 nor 2

Answer: (a) Explanation: Various provisions of the Indian Councils Act, 1861  Expansion of executive council of Governor General / Viceroy. The executive council of Governor General was added a fifth finance member. For legislative purpose, a provision was made for an addition of 6 to 12 members to the central executive. At least half of the additional members were to be non-officials. These members were nominated by the Viceroy for the period of two years. Further, the Governor General / Viceroy had been given some more powers such as:  He was authorized to nominate a president to preside over the meetings of the Executive council in his absence.  He had the power of making rules and regulations for the conduct of business of executive council.  He could create new provinces for legislative purposes and to appoint Lieutenant Governors for them. He was also empowered to alter, modify or adjust the limits of the provinces.  He could promulgate ordinances, without the concurrence of the legislative council, during and emergency.  Though the central council was empowered to legislate on all subjects concerning all persons and courts in British India but every bill passed required the assent of viceroy.  He could withhold his assent or exercise his veto power if he felt that the bill affected the safety, peace and interest of British India.  He had to communicate all laws to secretary of state for India who could disallow them with the assent of the crown.  Introduction of Portfolio System  Process of Decentralization No distinction between Central and Provincial subjects

49. A nationwide campaign “Darwaza Band” aims at: (a) Stopping women harassment (b) Promoting use of toilets

21

(c) Stresses on neighbourhood security (d) mooting a National cyber-security program for government installations

Answer: (b) Explanation: A nationwide campaign “Darwaza Band” was launched by the Ministry of Drinking Water and Sanitation, which is aimed at promoting use of toilets and discouraging open defecation. The campaign will run under the Swachh Bharat Mission. It has been supported by the World Bank.

50. Recently, an island in the has vanished due to coastal erosion. The name of this now inundated island is: (a) Androth (b) (c) (d) Parali I

Answer: (d) Explanation: One of the uninhabited islands that is a part of Lakshadweep has vanished due to coastal erosion and another four such territories in the sea are shrinking fast, claims a new study. Parali I Island, part of Bangaram , which had an area of 0.032 sq. Km in 1968 has been completely eroded, resulting in its inundation.

51. With reference to the “Rashtriya Krishi Vikas Yojana (RKVY)”, consider the following statements: (1) The scheme was originally launched in 2007 to achieve 4% annual growth in agricultural sector during 11thfive-year plan. (2) The scheme has been extended by three years i.e. 2017-18 to 2019-20and has been renamed as RKVY- RAFTAAR. Which of the statements given above is/are correct? (a) 1 only (b) 2 only (c) Both 1 and 2 (d) Neither 1 nor 2

Answer: (c) Explanation: About Rashtriya Krishi Vikas Yojana (RKVY): It was launched in 2007 on the recommendation of the National Development Council with an aim to achieve 4% annual growth in agriculture sector. Recently, government approved the continuation of Rashtriya Krishi Vikas Yojana (RKVY) as Rashtriya Krishi Vikas Yojana- Remunerative Approaches for Agriculture and Allied Sector Rejuvenation (RKVY- RAFTAAR) for three years i.e. 2017-18 to 2019-20.

52. In the context of Ashgabat Agreement, consider the following statements: (1) It is a recently concluded agreement to counter ISIS in Central Asia. (2) India is a founding member of the Ashgabat Agreement. Which of the statements given above is/are correct? (a) 1 only (b) 2 only (c) Both 1 and 2 (d) Neither 1 nor 2

Answer: (d) Explanation: The Ashgabat agreement is a multimodal transport agreement between India, Iran, Pakistan, Kazakhstan, Oman, Turkmenistan and Uzbekistan for creating an international transport and transit corridor facilitating transportation of goods between Central Asia and the Persian Gulf. The agreement came into force in April, 2016. The agreement was signed by Iran, Oman, Qatar, Turkmenistan and Uzbekistan on April 25, 2011. Qatar subsequently withdrew from the agreement in 2013. Kazakhstan joined the grouping in 2016. India formally joined Ashgabat agreement on 2nd February 2018.Turkmenistan is the depository state for the agreement.

22

53. Consider the following pairs:

Type of rocks Example 1. Igneous : Granite 2. Sedimentary : Chalk 3. Metamorphosis : Shale

Which of the pairs given above is/are correctly matched? (a) 1 and 2 only (b) 2 only (c) 2 and 3 only (d) 3 only

Answer: (a) Explanation: Igneous rocks examples - Granite, gabbro, pegmatite, basalt, etc. Sedimentary rocks examples - sandstone, conglomerate, limestone, shale, loess, geyserite, chalk, coal etc. Metamorphosis rocks examples - Gneissoid, slate, schist, marble, quartzite etc.

54. The August Offer was a proposal made by the British Government in 1940. In this context, which of the following statements is NOT correct? (a) Without the consent of minorities no future Constitution could be adopted. (b) The Muslim league had accepted the offer as it had tacitly approved the demand of Pakistan. (c) It provided for dominion status as an objective of British Policy for India. (d) It proposed to appoint a Constitution making body after the end of Second World War.

Answer: (b) Explanation: The august offer was a proposal made by the British government in 1940 promising the expansion of the Executive Council of the Viceroy of India to include more Indians, the establishment of an advisory war council, giving full weight to minority opinion, and the recognition of Indians' right to frame their own constitution (after the end of the war). However, this proposal was denied by the Congress as the minorities especially the Muslim league was assured that no constitutional scheme was acceptable to the government without their assurance. The Muslim league did not accept the offer as it did not give a clear assurance for the establishment of Pakistan. It offered to set up a Constituent Assembly after war. Jawahar Lal Nehru said that the whole idea was “dead and doornail”.

55. With reference to surface tension, consider the following statements: (1) The pressure inside a spherical drop is more inside as compared to outside. (2) In a liquid-air interface, the pressure will be higher on the convex side than on the concave side. Which of the statements given above is/are correct? (a) 1 only (b) 2 only (c) Both 1 and 2 (d) Neither 1 nor 2

Answer: (c) Explanation: (1) The pressure inside a spherical drop is more inside as compared to outside which gives it a spherical structure. (2) In a liquid-air interface, the pressure will be higher on the convex side than on the concave side. Take the example of bubbles. The pressure is more on the convex i.e. outside, keeping the bubbles intact until it touches an external surface.

56. With reference to the “Hydrocarbons Exploration and Licensing Policy (HELP)”, consider the following statements: 23

(1) It provides independent license for different types of hydrocarbons such as oil, gas, coal bed methane, etc. (2) Exploration phase for the onshore areas have been decreased from 7 to 5 years. Which of the statements given above is/are correct? (a) 1 only (b) 2 only (c) Both 1 and 2 (d) Neither 1 nor 2

Answer: (d) Explanation: 4 main features of HELP are: Uniform License: IT provides for a uniform licensing system to cover all hydrocarbons such as oil, gas, coal bed methane, etc. under a single licensing framework, instead of the present system of issuing separate licenses for each kind of hydrocarbons.

57. With reference to the “Independent Debt Management Office”, suggested by the NITI Aayog, consider the following statements: (1) Presently the RBI performs dual roles as Banker and Manager of Central Government’s borrowings. (2) According to NITI Aayog, it could lead to substantial reduction in India’s interest payment. Which of the statements given above is/are correct? (a) 1 only (b) 2 only (c) Both 1 and 2 (d) Neither 1 nor 2

Answer: (c) Explanation: Niti Aayog made a strong case for setting up an independent debt management office, saying better servicing of loans could lead to substantial reduction in India’s interest payment. At present, the government debt, including market borrowing, is managed by the Reserve Bank of India.

58. Consider the following folk dances of India:

Type of Folk Performed Dance predominantly in 1. Jhoomar : Punjab 2. Kummi : Tamil Nadu 3. Changu : Odisha

Which of the pairs given above is/are correctly matched? (a) 1 only (b) 1 and 2 only (c) 2 and 3 only (d) 1, 2 and 3

Answer: (d) Explanation: Jhoomar is a folk dance performed during the harvest season in Punjab. It is slower and more rhythmic form of bhangara. The Jhoomar is a dance of ecstasy. The dance is performed in circle to the tune of emotional songs. Kummiis the folk dance popular in Tamil Nadu. This is performed by women. This dance is usually performed during temple festivals, Pongal, family functions etc. Changu dance is a popular dance form from Orissa. It derives its name from a kind of drum called Changu, which invariably accompanies the dance. The dance is performed only by women.

24

59. The Bogibeel rail-road bridge is a double-deck bridge with a two-line railway track on the lower deck and a three-lane road on the upper. The 4.94km bridge is one of the longest river bridges in the country on the river (a) Brahmaputra (b) Ganges (c) Narmada (d) Krishna

Answer: (a) Explanation: The Bogibeel rail/road bridge is being built across the River Brahmaputra in the Dibrugarh district, in the state of Assam in north-east India. The 4.94km bridge is one of the longest river bridges in the country. The rail-road bridge is being undertaken by the Northeast Frontier Railway, which is a part of Indian Railways. The work on the project began in 2002 and was expected to be completed in seven years. But the bridge project experienced significant time and cost overruns before its construction gained momentum in July 2014. The project is now scheduled to be completed by March 2018.

60. Consider the following statements regarding the Environmental Performance Index: (1) It is a biennial index produced jointly by Yale University and Columbia University in collaboration with the World Economic Forum. (2) India ranked 117 out of 180, but still ranks higher than China at 120. Which of the statements given above is/are correct? (a) 1 only (b) 2 only (c) Both 1 and 2 (d) Neither 1 nor 2

Answer: (a) Explanation: Environmental Performance Index  It is biennial index produced jointly by Yale University and Columbia University in collaboration with the World Economic Forum.  The EPI ranks countries on 24 performance indicators across 10 issue categories.  Switzerland is ranked first followed by France and Denmark The 2018 Report  India fell 36 points lower as compared to 141 rank in 2016. China (120) and India (177) indicates strain population pressures and rapid economic growth impose on the environment.

61. “Clear skies and high temperatures; hot, dry summers and cool, wet winters. Climate is not extreme because of cooling from water bodies”. Which of the following cities may have such a climate? (1) Palermo (2) Cape Town (3) Santiago (4) Perth Select the correct answer using the code given below: (a) 1 and 3 only (b) 3 and 4 only (c) 2 and 4 only (d) 1, 2, 3 and 4

Answer: (d) Explanation: The climate being talked about is the Mediterranean Climate Clear skies and high temperatures; hot, dry summers and cool, wet winters. 25

 Mean annual precipitation ranges from 35 – 90 cm.  Temperature of warmest month greater than or equal to 10⁰ C.  Temperature of coldest month is less than 18⁰ C but greater than –3⁰ C  Climate is not extreme because of cooling from water bodies.

62. In the context of the boycott of Simon Commission in 1927, consider the following statements: (1) The Muslim League under Jinnah and the liberals of Hindu Mahasabha had not supported the anti-Simon Commission Movement. (2) Mahatma Gandhi was not involved in anti-Simon Commission Movement. (3) The Nehru Report of 1928 was an alternative constitutional reform to that of Simon Commission. Which of the statements given above is/are correct? (a) 1, 2 and 3 (b) 2 and 3 only (c) 1 and 2 only (d) 1 and 3 only

Answer: (b) Explanation: At its Madras session in 1927 presided over by Dr. Ansari, the National Congress passed a resolution which advocated the boycott of the Simon Commission “at every stage and in every form.“ The Muslim League and Hindu Mahasabha decided to support the Congress decision. As a gesture of solidarity with the nationalists, the Muslim League even accepted the principle of joint electorates, provided seats were reserved for the Muslims. Mahatma Gandhi during anti-Simon Commission demonstrations was not convinced that the time for mass political struggle had come yet. The rejection by Indian leaders of the all- white Simon Commission led Lord Birkenhead, the Secretary of State for India to make a speech in the House of Lords in which he challenged the Indians to draft a Constitution implying that they could not produce one that would be widely acceptable among the leaders of the various Indian communities.

63. In which of the following circumstances the Chief Minister of a State is NOT eligible to vote in the Presidential election? (1) If the Chief Minister is a caretaker Chief Minister. (2) If the Chief Minister is yet to prove his majority on the floor of the Lower House of the State Legislature. Select the correct answer using the code given below: (a) 1 only (b) 2 only (c) Both 1 and 2 (d) Neither 1 nor 2

Answer: (d) Explanation: The President is elected by the members of an electoral college consisting of the elected members of both the Houses of Parliament and the elected members of the Legislative Assemblies of States and the Union Territories of Delhi and Pondicherry. Nominated members to the two houses don’t participate in the election to the office of the President of India. It should be noted here that nominated members do participate in the impeachment of President of India. Also if the Chief Minister is a member of the Upper House of the State legislature then he is not eligible to vote in the Presidential election.

64. Consider the following statements regarding shell companies: (1) The Companies Act, 2013 has defined what a ‘shell company’ is and gives guideline to identify a shell company. (2) The Double Tax Avoidance Agreement (DTAA) is used sometimes as loophole by shell companies to avoid tax. Which of the statements given above is/are correct? (a) 1 only (b) 2 only (c) Both 1 and 2 26

(d) Neither 1 nor 2

Answer: (b) Explanation: What are shell companies? The Companies Act, 2013 has not defined what a ‘shell company’ is and as to what kind of activities would lead to a company being termed a ‘shell’. Shell companies are typically corporate entities which do not have any active business operations or significant assets in their possession. The government views them with suspicion as some of them could be used for money laundering, tax evasion and other illegal activities. How to strike off a shell company from the records? Companies can be removed from the rolls of the Ministry of Corporate Affairs by two means: strike off by Registrar of Companies (RoC) — (Section 248 (1) of the Companies Act, 2013) and voluntary strike off — (Section 248 (2) of the Companies Act, 2013). Voluntary closure can be done with the approval of the board and shareholders and the firm should have nil liabilities. Scenarios leading to the Strike off: The strike off happens in case of companies which have failed to commence business within a year of incorporation. Also, in case of companies that are not carrying on any business or operation for a period of two immediately preceding financial years and have not made any application within such period for obtaining the status of a ‘dormant company’ under Section 455 of the Companies Act can be struck off by the RoC unless cause is shown to the contrary. The RoC issues a show-cause notice to such companies and their directors seeking their response within 30 days. If the response is not satisfactory, the company’s name would be removed from the register. India recently amended its Double Taxation Avoidance Agreement (DTAA) with Mauritius to plug certain loopholes. Now, a Mauritian entity will have to pay capital gains tax here while selling shares in a company in India from April 2017. Earlier, the company could avoid tax as it was not a ‘resident’ in India. It could get away from the taxman in Mauritius too, due to non-taxation of capital gains for its residents. As a result, many shell entities sprang up in Mauritius to profit from investments in India and get away without paying taxes anywhere.

65. In the context of which of the following do you sometimes find the terms “Market Economy Status”, and “Most Favoured Nation” in the news? (a) WTO affairs (b) Affairs related to Bilateral Investment treaty (c) World Bank affairs (d) Affairs related to Double Taxation Avoidance Agreements

Answer: (a) Explanation: Market Economy Status and Most favoured nation are the terms associated with WTO Affairs.

66. Arrange the following ecosystem types in increasing order of their net primary productivity: (1) Temperate Grasslands (2) Savanna (3) Swamps and marshes (4) Boreal or Taiga Forests Select the correct answer using the code given below: (a) 1-2-3-4 (b) 4-1-2-3 (c) 2-4-1-3 (d) 4-2-1-3

Answer: (b) Explanation: Primary productivity is the rate of conversion of solar energy into plant matter. The rate of solar energy conversion into carbohydrates (total photosynthesis) is gross primary productivity (GPP). However, a portion of gross primary productivity must be expended by the plant through metabolic processes

27

necessary for maintenance, growth, and reproduction, and is lost as heat through respiration. The net rate at which energy is converted to carbohydrates is net primary productivity. The energy stored as organic matter (net primary production or NPP) becomes available to heterotrophs.

67. Consider the following leaders: (1) Mahatma Gandhi (2) Muhammad Ali Jinnah (3) Jawaharlal Nehru (4) S. Subramanian Iyer Which of the above leaders were associated with the Indian Home Rule League? (a) 1 only (b) 2 and 3 only (c) 2 and 4 only (d) 1, 2, 3 and 4

Answer: (d) Explanation: Between 1916 and 1918 when the First World War was beginning, prominent Indians like Joseph Baptista, Muhammad Ali Jinnah, Bal Gangadhar Tilak, G.S. Khaparde, Sir S. Subramanian Iyer and the leaders of the Theosophical Society Annie Besant decided to organise a national alliance of leagues across India, specifically to demand Home Rule or Self Government within the British Empire for all of India. Tilak founded the first Home Rule League at the Bombay Provincial Congress at Belgaum in April 1916, then after this Annie Besant founded Second League at Adyar Madras in September 1916. Jawaharlal Nehru joined Annie Besant’s league. The All India Home Rule League elected Mahatma Gandhi as its president in 1920. After that within a year it merged into the Indian National Congress.

68. Which of the following is/are the correct statements regarding ‘IceCube Observatory’ that was in the news recently? (1) It is the world’s most sensitive neutrino telescope which will help demystify the powerful pulses of radio energy generated up to billions of light-years away from Earth. (2) The Observatory has been constructed on a lonely peak in the western Himalayas and is the world’s highest observatory. Select the correct answer using the code given below: (a) 1 only (b) 2 only (c) Both 1 and 2 (d) Neither 1 nor 2

Answer: (a) Explanation: IceCube is a particle detector at the South Pole that records the interactions of a nearly mass less subatomic particle called the neutrino. IceCube searches for neutrinos from the most violent astrophysical sources: events like exploding stars, gamma-ray bursts, and cataclysmic phenomena involving black holes and neutron stars. The IceCube telescope is a powerful tool to search for dark matter and could reveal the physical processes associated with the enigmatic origin of the highest energy particles in nature.

69. Which of the following could be one of the reasons behind the fact that the weight of an individual is greater at the poles than at the equator? (a) Earth’s axis is tilted 23.5 degrees from the plane of its orbit around the Sun. (b) The gravitational pull of the Moon is maximum at the equator. (c) Difference between centrifugal force at equator and at poles. (d) Coriolis force which is maximum at equator and zero at poles.

Answer: (c) Explanation: The weight of an individual is greater at the poles than at the equator because at the equator the centrifugal force due to the spinning of the Earth is at its maximum and vanishes at the poles. This means that the attractive force of gravity is slightly reduced because it is directed towards the centre of the Earth, while the centrifugal force is directed outward from the centre. The effective acceleration of gravity at

28

the poles is 980.665 cm/sec/sec while at the equator it is 3.39 cm/sec/sec less due to the centrifugal force. If you weighed 100 pounds at the north pole on a spring scale, at the equator you would weigh 99.65 pounds, or 5.5 ounces less. Your mass, in grams, however would stay the same because 'grams' is a measure of the resistance of a body to being moved and has nothing to do with acceleration or gravity. Your mass in kilograms would remain the same. It is common for people to use 'pounds' and 'grams' interchangeably but they are not.

70. Consider the following statements regarding the Kigali Agreement: (1) It amends the 1987 Montreal Protocol to include gases responsible for global warming and will be binding on countries from 2019. (2) India forms part of the third group, wherein it has to reduce its hydro fluorocarbon (HFC) usage to about 15% of 2012 levels by 2025 and has to put a complete stop to HFC usage by 2028. Which of the statements given above is/are correct? (a) 1 only (b) 2 only (c) Both 1 and 2 (d) Neither 1 nor 2

Answer: (a) Explanation: 197 countries have struck a new landmark deal at Kigali in Rwanda to reduce the emissions of category of greenhouse gases (GHGs) which leads to hydro fluorocarbons (HFCs) Significance It amends the 1987 Montreal Protocol to include gases responsible for global warming and will be binding on countries from 2019. It also provides for penalties for non-compliance. It has divided the signatory parties into three groups -  The first group consists of rich and developed economies like USA, UK and EU countries who will start to phase down HFCs by 2019 and reduce it to 15% of 2012 levels by 2036.  The second group consists of emerging economies like China, Brazil as well as some African countries who will start phase down by 2024 and reduce it to 20% of 2021 levels by 2045.  The third group consists of developing economies and some of the hottest climatic countries like India, Pakistan, Iran, Saudi Arabia who will start phasing down HFCs by 2028 and reduce it to 15% of 2024- 2026 levels till 2047.

71. Consider the following statements regarding PPP investment models in India: (1) In a DBFO (Design, Build, Finance and Operate), the facility is owned by the private sector for the contract period and it recovers costs through public subvention. (2) This type of investment is particularly suited to roads, water and waste projects. Which of the statements given above is/are correct? (a) 1 only (b) 2 only (c) Both 1 and 2 (d) Neither 1 nor 2

Answer: (c)

29

Explanation:

72. In the context of Medieval Indian History, which of the following statements is/are correct regarding the Vijayanagar empire? (1) The unique feature of the Vijayanagar empire was the absence of caste system. (2) Ashtapradhan was a council of ministers in the Vijayanagar administration. (3) Portuguese general Afonso de Albuquerque captured Goa from the ruler of Vijayanagar kingdom. Select the correct answer using the code given below: (a) 1 and 2 only (b) 3 only (c) 1, 2 and 3 (d) None of the above

Answer: (d) Explanation: Portuguese general Afonso de Albuquerque captured Goa from the ruler of Bijapur. Caste system prevailed during the Vijayanagara Empire and while Brahmins followed the traditional profession of teachers and priests, Vaishyas were generally merchants. While the Sudras were mainly involved in agricultural and other works, Kshatriya Varna seems to be absent during the empire. The Ashta Pradhan was a council of eight ministers that administered the Maratha empire. The council was formed in 1674 by, Chatrapati Shivaji Maharaj.

73. With reference to “INS Karanj”, which was in the news, consider the following statements: (1) It is the first Indian submarine which can be operated only in the tropical climate. (2) It has been built with the help of French naval defence and energy company DCNS. (3) It is equipped with the Air Independent Propulsion System which enables it to stay underwater for longer duration. Which of the statements given above is/are correct? (a) 1 and 2 only (b) 2 only (c) 1 and 3 only (d) 3 only

Answer: (b) Explanation: Indian Navy has launched Scorpene submarine INS Karanj at the Mazagon Dock Shipbuilders Ltd (MDL), Mumbai, Maharashtra. In all, six scorpion submarines being indigenously built at MDL under Proje 75 with help of French naval defence and Energy Company DCNS. The first of these, INS Kalvari was commissioned in December 2017 and second, INS Khanderi is undergoing sea trials. Remaining 4 submarines will be inducted gradually by 2020. These are diesel-electric attack submarines equipped with anti-ship missile. The first four submarines will be conventional, while last two will be 30

equipped with the Air Independent Propulsion (AIP) system, which will enable them to stay underwater for longer duration.

74. With reference to the difference between the No Confidence Motion and Censure Motion, which of the following statements is/are correct? (1) Both Censure Motion and No-confidence motion can be moved only in the Lok Sabha. (2) In case of No-confidence Motion reasons must be stated for its adoption while in case of Censure motion no such reasons for its adoption are required. (3) Government will collapse if both the Censure motion and the No Confidence Motion are passed in the Lok Sabha. Select the correct answer using the code given below: (a) 1 only (b) 2 and 3 only (c) 1 and 3 only (d) 1, 2 and 3

Answer: (a) Explanation: In case of Censure motion, reasons must be stated for its adoption in the Lok Sabha while in case of motion of no confidence no such reasons for its adoption are required to be stated in the Lok Sabha. The council of ministers need not resign from the office if censure motion is passed in the Lok Sabha while in case of motion of no confidence, the council of minister must resign from office if the motion is passed.

75. In the context of Nitrogen Cycle, consider the following: Soil Nitrates are transformed into free Nitrogen by which of the following Bacteria? (a) Denitrifying bacteria (b) Nitrifying bacteria (c) Ammonifying bacteria (d) None of the above

Answer: (a) Explanation: Denitrifying bacteria are a diverse group of bacteria together with denitrifying fungi and archaea, is capable of performing denitrification as part of the nitrogen cycle. They metabolise nitrogenous compounds using various enzymes, turning nitrogen oxides back to nitrogen gas or nitrous oxide.

76. What is “GeM 3.0”, recently in the news? (1) It is an online platform to provide information on Genetically Modified Crops in India. (2) It functions under the Ministry of Environment, Forests and Climate Change. Which of the statements given above is/are correct? (a) 1 only (b) 2 only (c) Both 1 and 2 (d) Neither 1 nor 2

Answer: (d) Explanation: The Ministry of Commerce & Industry has launched third version of the Government e- Marketplace (GeM 3.0). The earlier version GeM 2.0 was launched as pilot project in August 2016. GeM is an Online Market platform to facilitate procurement of goods and services by various Ministries and agencies of the Government. It functions under Directorate General of Supplies and Disposals (DGS&D), Ministry of Commerce and Industries. It is completely paperless, cashless and system driven e-market place that enables procurement of common use goods and services with minimal human interface.

77. Hindu Philosophy is traditionally divided into six astika (orthodox) schools of thought, or darsanam, which accept the Vedas as supreme revealed scriptures. In this context, consider the following astika schools:

31

Astika Schools Characteristics 1. Samkhya : An atheistic and strongly dualist theoretical exposition of consciousness and matter 2. Vaisheshika : An empiricist school of atomism 3. Mimamsa : A school emphasizing meditation, contemplation and liberation

Which of the above is/are correctly matched? (a) 1 only (b) 1 and 2 only (c) 2 and 3 only (d) 1, 2 and 3

Answer: (b) Explanation: Hindu philosophy is traditionally divided into six āstika (orthodox) schools of thought, or darśanam, which accept the Vedas as supreme revealed scriptures. The āstika schools are:  Samkhya, an atheistic and strongly dualist theoretical exposition of consciousness and matter.  Yoga, a school emphasizing meditation, contemplation and liberation.  Nyayaor logic, explores sources of knowledge (NyāyaSūtras).  Vaisheshika, an empiricist school of atomism.  Mimāṃsā, an anti-ascetic and anti-mysticist school of orthopraxy.  Vedanta, the last segment of knowledge in the Vedas, or the 'Jnan' (knowledge) 'Kanda' (section). Vedanta came to be the dominant current of Hinduism in the post-medieval period. Of the historical division into six darsanas, only two schools, Vedanta and Yoga, survive.

78. Consider the following statements regarding the different price indices prepared in India: (1) Wholesale Price Index (WPI) is prepared by the Office of Economic Affairs, Ministry of Commerce and Industries (MOCI). (2) Consumer Price Index- Industrial Worker (CPI-IW) is prepared by the Ministry of Commerce and Industry. (3) CPI-All India is prepared by CSO, Ministry of Statistics and Programme Implementation (MOSPI). Which of the statements given above is/are correct? (a) 1 and 2 only (b) 2 and 3 only (c) 1 and 3 only (d) 1, 2 and 3

Answer: (c) Explanation:

Index Agency Base Year WPI Office of Economic Affairs, Ministry of Commerce and 2004-05 Industries (MOCI) CPI All India, CPI -Urban and Rural CSO, Ministry of Statistics and Programme 2012 Implementation (MOSPI) CPI-AL Labour Bureau, Ministry of Labour and Employment 1986-87 CPI-RL (MOLE) 1986-87 CPI-IW 2001

79. Consider the following statements: (1) Gravitational force is the weakest among the four fundamental forces, viz. Gravitational, Electromagnetic, Strong and Weak forces. (2) Conduction requires physical contact, whereas convection and radiation do not need it. 32

Which of the statements given above is/are correct? (a) 1 only (b) 2 only (c) Both 1 and 2 (d) Neither 1 nor 2

Answer: (c) Explanation:

80. A joint initiative taken by the Assam State Government and the Armed Forces under the guidelines of National Disaster Management Policy 2016, is better known by the name of (a) Exercise Jalrahat (b) Exercise Malhar (c) Exercise Prakampana (d) Exercise Sahayata

Answer: (a) Explanation: Exercise jalrahat is a joint initiative taken by the Assam state government and Armed forces towards the objective of improving disaster preparedness during floods. Similar exercises will be done in  Bhuj, Gujarat by Indian Air Force- Exercise Sahayata for major earthquake.  Vishakhapatnam by Indian Navy- Exercise Prakampana for preparedness in case of super cyclone

81. With reference to the National Testing Agency (NTA), which of the following statements is/are correct? (1) NTA would be an autonomous and self-sustained premier testing organisation registered under the Indian Society Registration Act, 1860. (2) It will conduct a national competition called the Teaching Professionals’ Olympiad and offer a Certification of competence for teachers. Select the correct answer using the code given below: (a) 1 only (b) 2 only (c) Both 1 and 2 (d) Neither 1 nor 2

Answer: (a) Explanation: Cabinet has approved National Testing Agency (NTA) as a Society registered under the Indian Societies Registration Act, 1860. It is as an autonomous and self-sustained premier testing organization to conduct entrance examinations for higher educational institutions. The NTA would initially conduct those entrance examinations which are currently being conducted by the CBSE. Other examinations will be taken up gradually after NTA is fully geared up. The entrance examinations will be conducted in online mode at least twice a year, thereby giving adequate opportunity to candidates to bring out their best.

82. The Eight fold path of Buddhism, also called the Middle Path is the theme of which of the following? (a) Dipavamsa

33

(b) Divyavadana (c) Mahaparinibbana Sutta (d) Dharma Chakra Pravartana Sutta

Answer: (d) Explanation: (1) The Dipavamsa is the oldest historical record of Sri Lanka. It means chronicle of the Island. The chronicle is believed to be compiled from Attakatha and other sources around the 3-4th century. (2) The Divyavadana is a sanskrit anthology of Buddhist tales. It may be dated to 2nd century CE. (3) The Mahaparinibbana Sutta is 16th Sutta in the Digha Nikaya, a scripture belonging the Sutta Pataka of Theravada Buddhism. It concerns the end of Gautam Buddha’s life and is the longest sutta of the Pali canon.

83. Which of the following are matters on which a Constitutional Amendment is possible only when the Bill is passed in each House by a majority of the total membership of that House and by a majority of not less than two thirds of the members of that House present and voting? (1) Admission or establishment of new States (2) Fundamental Rights (3) Directive Principles of State Policy (4) Delimitation of constituencies Select the correct answer using the code given below: (a) 1, 2 and 3 only (b) 2 and 3 only (c) 1 and 3 only (d) 1, 2, 3 and 4

Answer: (b) Explanation: The majority of the provisions in the Constitution need to be amended by a special majority of the Parliament, that is, a majority (that is, more than 50 per cent) of the total membership of each House and a majority of two-thirds of the members of each House present and voting. The expression ‘total membership’ means the total number of members comprising the House irrespective of fact whether there are vacancies or absentees. The provisions which can be amended by this way includes: (i) Fundamental Rights; (ii) Directive Principles of State Policy; and (iii) All other provisions which are not covered by the first and third categories.

84. With reference to Humidity, which of the following statements is correct? (a) Humidity of the air does not get affected by the change in atmospheric temperature. (b) Humidity of the air increases with the increase in atmospheric temperature. (c) Humidity of the air does not show any consistent behaviour with the change in atmospheric temperature. (d) Humidity of the air decreases with the increase in atmospheric temperature.

Answer: (b) Explanation: Humidity is the amount of water vapour present in the air. Water vapour is the gaseous state of water and is invisible to the human eye. Humidity indicates the likelihood of precipitation, dew, or fog. Increasing the temperature speeds up evaporation and thereby shifts the balance further toward water vapour, so the higher the temperature, the more moisture the air must contain before it's saturated.

85. With reference to the Draft Guidelines for Stem Cell Research 2017, consider the following statements: (1) The Indian Council of Medical Research (ICMR) in association with the Department of Biotechnology drafted these guidelines. (2) It prohibits areas for stem cell research including human germ line gene therapy and reproductive cloning. Which of the statements given above is/are correct? (a) 1 only (b) 2 only

34

(c) Both 1 and 2 (d) Neither 1 nor 2

Answer: (c) Explanation: The Indian Council of Medical Research (ICMR) in association with the Department of Biotechnology recently released the revised Draft Guidelines for Stem Cell Research, 2017 for comments. Permissible Areas of Research include, In vitro studies using stem cells isolated from tissues, with prior approval. Prohibited Areas of Research: In the current state of scientific knowledge and understanding, stem cell research in the following areas is prohibited: 1. Research related to human germ line gene therapy and reproductive cloning. 2. In vitro culture of intact human embryos, regardless of the method of their derivation, beyond 14 days of fertilization or formation of primitive streak, whichever is earlier. 3. Clinical trials involving xenogeneic cells. 4. Any clinical research on Xenogeneic-Human hybrids.

86. Recently Rotavac became the first indigenously developed vaccine from India to be pre-qualified by World Health Organisation (WHO).In this context, consider the following statements: (1) Rotavac was developed under the joint collaboration between India, Israel and UNICEF. (2) Rotavirus is the most common causative agent of moderate to severe diarrhoea among infants below 5 years age group in India. Which of the statements given above is/are correct? (a) 1 only (b) 2 only (c) Both 1 and 2 (d) Neither 1 nor 2

Answer: (b) Explanation: Rotavac became first ingeniously developed vaccine from India to be pre-qualified by World Health Organization (WHO). It means that vaccine can be sold internationally to several countries in South America and Africa. So far, several vaccines from India have been pre-qualified by WHO, but Rotavac is the first vaccine entirely developed locally to get this status in safety and efficacy. Rotavac conceived and developed by the Hyderabad-based Bharat Biotec Limited. It protects against childhood diarrhoea caused by the rotavirus. It was developed under the joint collaboration between India and United States in area of medical research. It was developed under public- private partnership (PPP) model that involve Ministry of Science and Technology, institutions of the US Government and NGOs in India supported by Bill and Melinda Gates Foundation. It is a most common causative agent of moderate-to-severe diarrhoea (MSD) among infants and children below 5 years of age in India. It spreads from person to person due to bacterial and parasiting agents that are primarily transmitted through contaminated food or water.

87. The principle of Anekantavada or Syadvada is a very valuable contribution by an ancient religion to the world thought. In this context consider the following statements: (1) This principle teaches us how to realise truth in its varied aspects. (2) It is one of the most important and fundamental doctrine of Buddhism. Which of the statements given above is/are correct? (a) 1 only (b) 2 only (c) Both 1 and 2 (d) Neither 1 nor 2

Answer: (a) Explanation: Anekantavada refers to the principles of pluralism and multiplicity of viewpoints, or vantage points, the notion that reality is perceived differently from diverse points of view, and that no single point of view is the

35

complete truth, yet taken together they comprise the complete truth. It is one of the most important and fundamental doctrine of Jainism. Syadavada is the Jain doctrine of epistemological relativism. It means all judgements are conditional, holding good only in certain conditions, circumstances, or senses, expressed by the word ‘syat’ (sanskrit: ‘may be’).

88. Recently severe Pink Bollworm attack has inflicted which of the following crops in various regions of India? (a) Cotton (b) Sunflower (c) Soyabean (d) Potato

Answer: (a) Explanation: Recently some States of India have witnessed pink bollworm (PBW) attacks on cotton, especially in Maharashtra and also in Telangana, Andhra Pradesh, Karnataka and Gujarat. The infestation of this insect pest — whose larvae bore into cotton bolls through the lint fibre to feed on the seeds — happened during October, just when the crop was maturing and almost ready for its first-flush pickings. The pink bollworm eats away the cotton fibre and the bolls, causing economic loss to the farmer.

89. A strait is a thin channel of a waterway that connects two large water bodies. In this context which of the following statements is NOT correct? (a) Hormuz strait joins the Gulf of Oman and the Persian Gulf. (b) Bab-el-Mandeb joins the Gulf of Aden and the Red Sea. (c) Sunda Strait joins the Java Sea and the . (d) Bering Strait joins the Atlantic Ocean and the Mediterranean Sea.

Answer: (d) Explanation: Bosphorus Strait divides Europe from Asia and joins Black Sea to Marmara Sea.

36

90. The CarbFix2 Project captures CO2 directly from ambient air, dissolves it in water and then pumps it into an injection site, where the CO2 reacts with basaltic bedrock, forming solid carbonate minerals. This project is functioning in: (a) Ireland (b) Greenland (c) Iceland (d) Denmark

Answer: (c) Explanation:  Reykjavik Energy’s Edda Aradóttir has launched the first negative emissions plant in the world. 37

 Located at the Hellisheidi Power Plant, the CarbFix2 project captures CO2 directly from ambient air. It then dissolves it in water and then pumps it into an injection site near the facility, where the CO2reacts with basaltic bedrock, forming solid carbonate minerals.  Not only does the project put a dent in global warming, it can also provide eco-friendly construction materials.

 CarbFix 2 is taking carbon sequestration to the next level by capturing CO2 directly from the ambient air. Essentially, this plant generates negative emissions.

91. Which of the following countries conducted the bi-annual multilateral exercise ‘Blue Flag-17’ in which India has participated for the first time? (a) Israel (b) United States of America (c) France (d) Germany

Answer: (a) Explanation: Blue Flag is a military aviation exercise held by the Israeli Air Force. It first took place in November 2013 at Ovda Air Force Base in Israel. The exercise, which included the participation of several foreign air forces, is aimed at expanding international cooperation. Plans on making "Blue Flag" a biennial event were realised in 2015 with another "Blue Flag" exercise in which the air forces of the United States, Greece and Poland also participated. In 2017 the exercise host air forces from the United States of America, Poland, Italy and Greece – and for the first time India, France and Germany have participated.

92. With reference to cryptocurrency, consider the following statements: (1) It is a peer-to-peer digital currency system which provides a high level of privacy protection. (2) It has a centralised model of operation, which makes its use more robust. Which of the statements given above is/are correct? (a) 1 only (b) 2 only (c) Both 1 and 2 (d) Neither 1 nor 2

Answer: (a) Explanation: Cryptocurrency is a peer-to-peer, decentralized and digital currency system that furnish online users with the capability of processing transactions using digital exchange units known as virtual currency. This system was created by the Japanese computer programmer known by the pseudonym of Satoshi Nakamoto in 2009. Benefits of cryptocurrency: 1. Privacy Protection. 2. Cost-effectiveness 3. Lower Entry Barriers 4. Alternative to Banking Systems and Fiat Currencies 5. Open Source Methodology and Public Participation 6. Immunity to Government led Financial Retribution Risks involved: 1. Distributed Denial of Service (DDoS) attacks on Cryptocurrency System 2. Uncertain Regulatory Environment 3. Lack of Liquidity and Lower Acceptability 4. Price Volatility 5. Uncertainty over Consumer Protection and Dispute Settlement Mechanisms

93. Which of the following is NOT mentioned in the Constitution of India but followed as a convention? (a) Appointment of the senior most judge as the Chief Justice of India. (b) All sections of India are to be represented in the Union Council of Ministers. (c) The Minister of Parliamentary Affairs is to be a member of the Lower House. (d) A candidate cannot stand for election from more than one Parliamentary Constituency.

38

Answer: (a) Explanation: Article 124 of the Constitution Of India states about the Supreme Court with a Chief Justice but to appoint whom the Constitution is silent. Hence, practice of appointing Senior most judge has became a practice. The relevant part of clause (2) of Article 124 reads:“Every judge of the Supreme Court shall be appointed by the president by warrant under his hand and seal after consultation with such of the judges of the Supreme Court and of the high courts in the states as the president may deem necessary for the purpose and shall hold office until he attains the age of 65 years. Provided that in the case of appointment of a judge other than the chief justice, the Chief Justice of India shall always be consulted.” Only in 1975 during emergency, this convention was not followed and govt. appointed CJI bypassing 3 senior most judges.

94. Consider the following elements: (1) Copper (2) Phosphorus (3) Manganese (4) Potassium (5) Zinc Which of the above micro-elements are needed to increase soil fertility? (a) 2 and 4 only (b) 1, 3, 4 and 5 only (c) 1, 3 and 5 only (d) 1, 2, 3, 4 and 5

Answer: (c) Explanation: The six macro-nutrients, nitrogen (N), phosphorus (P), potassium (K), calcium (Ca), magnesium (Mg), and sulphur (S) are required by plants in large amounts. The rest of the elements are required in trace amounts (micro-nutrients). Essential trace elements include boron (B), chlorine (Cl), copper (Cu), iron (Fe), manganese (Mn), sodium (Na), zinc (Zn), molybdenum (Mo), and nickel (Ni). Beneficial mineral elements include silicon (Si) and cobalt (Co).

95. In the context of the Treaty on the Prohibition of Nuclear Weapons, which of the following statements is NOT correct? (a) It is the first legally binding international agreement to comprehensively prohibit nuclear weapons. (b) In order for the treaty to come into effect the signature of and ratification by at least 50 countries is required. (c) The International Campaign to Abolish Nuclear Weapons helped bring about this treaty. (d) India has not actively participated in its negotiations and is the first country to ratify this treaty.

Answer: (d) Explanation: The Treaty on the Prohibition of Nuclear Weapons, or the Nuclear Weapon Ban Treaty, is the first legally binding international agreement to comprehensively prohibit nuclear weapons, with the goal of leading towards their total elimination. In the vote in the United Nations on the treaty text, 122 were in favour, 1 voted against (Netherlands), and 1 abstained (Singapore). 69 nations did not vote, among them all of the nuclear weapon states and all NATO members except the Netherlands. India did not participate in negotiations for a treaty in the prohibition of nuclear weapons at the United Nations, saying it did not contribute to the development of any customary international law. Therefore, question of ratifying does not arise.

96. Consider the following pairs:

Report Published by 1. Global Environment Outlook : World Wide Fund for Nature 2. Living Planet Report : United Nations Environment Programme 3. The Sustainable Development Goals Report : The United Nations

Which of the above pairs is/are correctly matched? 39

(a) 1 and 2 only (b) 2 and 3 only (c) 3 only (d) None of the above

Answer: (d) Explanation: Global Environment Outlook (GEO) is a series of reports on the environment issued periodically by the United Nations Environmental Programme (UNEP). The GEO project was initiated in response to the environmental reporting requirements of UN Agenda 21 and to a UNEP Governing Council decision of May 1995 which requested the production of a new comprehensive global state of the environment report. The Living Planet Report is published every two years by the World-Wide Fund for Nature since 1998. It is based on the Living Planet Index and ecological footprint calculations. The Living Planet Report is the world's leading, science-based analysis on the health of our only planet and the impact of human activity. Humanity's demands exceed the Earth's capacity to sustain us. The latest edition of the Living Planet Report was released in October 2016. The Sustainable Development Goals Report is released by Sustainable Development Solutions Network (SDSN) and Bertelsmann Stiftung; an independent foundation under private law, based in Gütersloh, Germany.

97. In the context of the powers of the Prime Minister of India, consider the following statements: (1) The Prime Minister is the leader of the majority party in both the Houses of Parliament. (2) The Prime Minister is free to choose his minister only from among members of either Houses of the Parliament. Which of the statements given above is/are correct? (a) 1 only (b) 2 only (c) Both 1 and 2 (d) Neither 1 nor 2

Answer: (d) Explanation: Leader of the Lok Sabha, the Lower House of the Indian Parliament, is the Prime Minister by default if he is a member of the Lok Sabha. If the Prime Minister is not a member of the Lower House of Parliament he can nominate another minister as the Leader of the House. A Minister (including Prime Minister) must be a member of any of the house of Parliament. If at the time of appointment, a minister or PM is not a member of any house, he must attain membership of any of them within 6 months.

98. With reference to the “National Cyber Coordination Centre” (NCCC), consider the following statements: (1) It is a multi-stakeholder body and comes under Indian Computer Emergency Response Team (CERT-In) at Ministry of Communications. (2) It derives its powers from the Information Technology Act, 2000. Which of the statements given above is/are correct? (a) 1 only (b) 2 only (c) Both 1 and 2 (d) Neither 1 nor 2

Answer: (b) Explanation:  NCCC is a multi-stakeholder cyber-security and e-surveillance agency  It comes under the Indian Computer Emergency Response Team (CERT-In), Union Ministry of Electronics and Information Technology  It has powers under the Indian constitution with provision of section 69B of the Information Technology Act, 2000  It will be India's first layer for cyber threat monitoring and all communication with the government as well as the private service providers will be monitored round the clock 40

 Its mandate is to scan internet traffic and communication metadata (which are little snippets of information hidden inside each communication) coming into the country to detect real-time cyber threats.

99. The Indian Space Research Organisation (ISRO) is planning to launch the first solar mission, Aditya- L1. Aditya-L1 mission is aimed at studying the Sun from an orbit around the Sun-Earth Lagrangian point 1 (L1). What is Lagrangian point? (a) It is a point on the outer layer of the Sun, extending to thousands of km above the disc (photosphere) having very high temperature. (b) It is the point where the combined gravitational force of two large bodies is equal to the centrifugal force that is felt by a third body which is relatively smaller (c) It is a point in near Sun at which the solar wind meets the interstellar medium or solar wind from other stars. (d) A point which is equidistant from the Sun and the Earth.

Answer - (b) Explanation – Aditya-L1, India’s maiden mission to the Sun, will be launched in 2019. The main aim of the solar mission is to improve our understanding of “dynamical processes of the sun,” and help resolve some outstanding questions in solar physics. The satellite carrying six payloads will be launched from Sriharikota in Andhra Pradesh on the PSLV- XL launch vehicle. It will be launched into the halo orbit around the Lagrangian point 1 (L1) of the Sun-Earth system. This orbit has the advantage of allowing continuous monitoring of the sun. ISRO has over the years led from the front to involve industry from the drawing board to the launch pad as a valued partner in many aspects of our space programme.

100. The first edition of the International Symposium to Promote Innovation & Research in Energy Efficiency (INSPIRE 2017) was held in Jaipur. Which of the following statements is/are correct about INSPIRE 2017? (1) It was organized by Energy Efficiency Services Limited (EESL) in partnership with The World Bank, and Alliance for an Energy Efficient Economy (AEEE). (2) It was a global conference which aims to bring together various stakeholders like policy makers, innovators, financiers to showcase best practices in the sector. Which of the statements given above is/are correct? (a) 1 only (b) 2 only (c) Both 1 and 2. (d) Neither 1 nor 2 Answer - (c) Explanation – The first edition of the International Symposium to Promote Innovation & Research in Energy Efficiency (INSPIRE 2017) was held in Jaipur. The five-day symposium is was organized by Energy Efficiency Services Limited (EESL) in partnership with The World Bank, and Alliance for an Energy Efficient Economy (AEEE). INSPIRE 2017 was an International Conference that brings together various stakeholders such as policy makers, innovators, financiers, influencers to showcase best practices in the sector.

41